1987 AJHSME Problems/Problem 3

Revision as of 15:04, 28 January 2009 by 5849206328x (talk | contribs) (New page: ==Problem== <math>2(81+83+85+87+89+91+93+95+97+99)=</math> <math>\text{(A)}\ 1600 \qquad \text{(B)}\ 1650 \qquad \text{(C)}\ 1700 \qquad \text{(D)}\ 1750 \qquad \text{(E)}\ 1800</math> ...)
(diff) ← Older revision | Latest revision (diff) | Newer revision → (diff)

Problem

$2(81+83+85+87+89+91+93+95+97+99)=$

$\text{(A)}\ 1600 \qquad \text{(B)}\ 1650 \qquad \text{(C)}\ 1700 \qquad \text{(D)}\ 1750 \qquad \text{(E)}\ 1800$

Solution

This problem needs a solution. If you have a solution for it, please help us out by adding it.

See Also

1987 AJHSME Problems